Difference between revisions of "2001 JBMO Problems/Problem 4"

(Created page with "__TOC__ ==Problem== Let <math>N</math> be a convex polygon with 1415 vertices and perimeter 2001. Prove that we can find 3 vertices of N which form a triangle of area smaller...")
 
Line 9: Line 9:
 
Well, then <math>2\sqrt{ab} < a+b (AM-GM) < 2\sqrt2 \Rightarrow ab < 2</math> and the area of the triangle with sides <math>a,b</math> and the angle <math>C</math> between them has area <math>\frac12ab\sin C \le \frac{1}{2}ab\sin \frac{\pi}{2} <\frac12\cdot2\cdot1 = 1</math> as desired.
 
Well, then <math>2\sqrt{ab} < a+b (AM-GM) < 2\sqrt2 \Rightarrow ab < 2</math> and the area of the triangle with sides <math>a,b</math> and the angle <math>C</math> between them has area <math>\frac12ab\sin C \le \frac{1}{2}ab\sin \frac{\pi}{2} <\frac12\cdot2\cdot1 = 1</math> as desired.
  
~ [[https://artofproblemsolving.com/community/user/61542 AwesomeToad]]
+
~ [https://artofproblemsolving.com/community/user/61542 AwesomeToad]
 +
 
 +
{{JBMO box|year=2001|num-b=2|after=Last Question}}

Revision as of 00:53, 9 January 2023

Problem

Let $N$ be a convex polygon with 1415 vertices and perimeter 2001. Prove that we can find 3 vertices of N which form a triangle of area smaller than 1.

Soultion

The largest side has length at least $\frac{2001}{1415}$. Therefore, the sum of the other $1414$ sides is $\frac{2001\cdot1414}{1415}$. Divide these sides into $707$ pairs of adjacent sides, and there exist one pair of sides $a,b$ such that $a+b \le \frac{2001\cdot1414}{1415\cdot707}=\frac{2001\cdot2}{1415}$. Blah blah blah... we know how to prove $\frac{2001\cdot2}{1415} < 2\sqrt2$ except why would a problem want you to do that.... no idea.

Well, then $2\sqrt{ab} < a+b (AM-GM) < 2\sqrt2 \Rightarrow ab < 2$ and the area of the triangle with sides $a,b$ and the angle $C$ between them has area $\frac12ab\sin C \le \frac{1}{2}ab\sin \frac{\pi}{2} <\frac12\cdot2\cdot1 = 1$ as desired.

~ AwesomeToad

2001 JBMO (ProblemsResources)
Preceded by
Problem 2
Followed by
Last Question
1 2 3 4 5
All JBMO Problems and Solutions